Question Stems Flashcards
If the statements above are true, which one of the following must also be true?
Must Be True
If the information above is correct, which one of the following conclusions can be properly drawn on the basis of it?
Must Be True
The statements above, if true, most strongly support which one of the following?
Must Be True
Which one of the following can be properly inferred from the passage?
Must Be True
Which one of the following is most strongly supported by the information above, if they are true?
Must Be True
Which one of the following inferences is most supported by the information above?
Must Be True
If all of the statements above are true, which one of the following must be true?
Must Be True
Which one of the following conclusions is most strongly supported by the information above?
Must Be True
Which one of the following most accurately expresses the main conclusion of the argument?
Main Point
Which one of the following most accurately expresses the conclusion of the journalist’s argument?
Main Point
Which one of the following most accurately restates the main point of the passage?
Main Point
The main point of the argument is that:
Main Point
Which one of the following most accurately expresses the argument’s conclusion?
Main Point
The X’s reasoning provides grounds for accepting which one of the following statements?
Must Be True
The information above provides the LEAST support for which one of the following?
Must Be True - X
The main point of the argument is that:
Main Point
Which one of the following, if true, most seriously weakens the argument?
Weaken
Which one of the following, if true, most undermines the X’s argument?
Weaken
Which one of the following, if true, would most call into question the analyst’s explanation of the price increase?
Weaken
Which one of the following, if true, could be used by X to counter Y’s rejection of her explanation?
Weaken
Which one of the following, if true, is the strongest logical counter parent P can make to parent Q’s objection?
Weaken
Which one of the following, if true, most calls into question the claim above?
Weaken
Which one of the following, if true, most seriously weakens the X’s argument?
Weaken
Which one of the following, if true, casts the most doubt on the conclusion above?
Weaken
Each of the following, if true, weakens the X’s argument EXCEPT:
Weaken - X
The reasoning in the advertisement would be most strengthened if which one of the following were true?
Strengthen
Each of the following, if true, would strengthen the X’s argument EXCEPT:
Strengthen - X
Which one of the following, if true, LEAST strengthens the argument above?
Strengthen - X
Which one of the following discoveries, if it were made, would most support the above hypothesis about X and Y?
Strengthen
Which one of the following, if true, would most strengthen the X’s argument?
Strengthen
Each of the following, if true, strengthens the X’s argument EXCEPT:
Strengthen - X
Which one of the following, if true, most supports the argument?
Strengthen
The conclusion above follows logically if which one of the following is assumed?
Justify the Conclusion
Which one of the following, if assumed, would allow the conclusion to be properly drawn?
Justify the Conclusion
Which one of the following, if assumed, enables the argument’s conclusion to be properly inferred?
Justify the Conclusion
Which one of the following is an assumption that would serve to justify the conclusion above?
Justify the Conclusion
The X’s conclusion would be properly drawn if it were true that the:
Justify the Conclusion
The conclusion above is properly drawn if which one of the following is assumed?
Justify the Conclusion
The X’s conclusion follows logically if which one of the following is assumed?
Justify the Conclusion
Which one of the following is an assumption required by the argument above?
Assumption
Which one of the following is an assumption upon which the argument depends?
Assumption
The argument assumes which one of the following?
Assumption
The conclusion in the passage above relies on which one of the following assumptions?
Assumption